หัวข้อ: USAMO 2005
ดูหนึ่งข้อความ
  #5  
Old 24 เมษายน 2005, 15:47
nongtum's Avatar
nongtum nongtum ไม่อยู่ในระบบ
ผู้พิทักษ์กฎทั่วไป
 
วันที่สมัครสมาชิก: 10 เมษายน 2005
ข้อความ: 3,246
nongtum is on a distinguished road
Post

อ้างอิง:
ข้อความเดิมของคุณ warut:
ผมก็สนใจข้อ 2 เหมือนกันครับ แต่พยายามคิดเท่าไหร่ก็ไม่ออกสักที ก็พอดีคุณ nongtum
มาตอบ แต่ไม่รู้เป็นไงมีตัวแปลกๆปนอยู่เต็มไปหมด อ่านยากมากเลยครับ แต่ผมก็พยายาม
ติดตามวิธีพิสูจน์ไปเรื่อยๆ ก็เข้าใจและเห็นด้วยมาตลอด แต่ตอนสุดท้ายนี่สิครับผมไม่แน่ใจ
ว่าคุณ nongtum เขียนอย่างนี้รึเปล่า\[x^3+1\equiv
(-1)\cdot\alpha_1+(-1)\cdot\beta_1\pmod4\]ถ้าใช่ผมว่าตรงนี้ไม่น่าจะถูกนะครับ จาก \(x^3+1=7^{\alpha_1}\cdot3^{\beta_1}\) เราควรจะได้ว่า\[x^3+1\equiv
(-1)^{\alpha_1+\beta_1}\pmod4\]มากกว่านะครับ ซึ่งถ้าที่ผมคิดต่อไปไม่ผิด นี่จะยังไม่เพียงพอที่จะพิสูจน์โจทย์ข้อนี้ได้ครับ
ขอบคุณมากครับสำหรับตำท้วงติง ตอนนี้เลยยังคิดไม่ออกเหมือนกัน คือคิดไปคิดมาทำท่าว่าจะไม่จบแค่ mod 8 เอาซะอย่างงั้น แต่ยังเชื่อว่าสามารถแก้ไดด้วยวิธีที่พิมพ์ให้ดู เอาเป็นว่าผมจะมาแก้อีกทีตอน'คิดเอง'ออกละกันครับ ขอหลบไปปั่นการบ้านที่กำลังท่วมหัวก่อน
________________________________
I love spoilers.
__________________
คนไทยร่วมใจอย่าใช้ภาษาวิบัติ
ฝึกพิมพ์สัญลักษณ์สักนิด ชีวิต(คนตอบและคนถาม)จะง่ายขึ้นเยอะ (จริงๆนะ)

Stay Hungry. Stay Foolish.
ตอบพร้อมอ้างอิงข้อความนี้